Putnam 2016/B6

Συντονιστής: Demetres

Άβαταρ μέλους
Demetres
Γενικός Συντονιστής
Δημοσιεύσεις: 8989
Εγγραφή: Δευ Ιαν 19, 2009 5:16 pm
Τοποθεσία: Λεμεσός/Πύλα
Επικοινωνία:

Putnam 2016/B6

#1

Μη αναγνωσμένη δημοσίευση από Demetres » Τετ Ιαν 04, 2017 8:44 pm

Να υπολογιστεί το

\displaystyle{\sum_{k=1}^{\infty}\frac{(-1)^{k-1}}{k}\sum_{n=0}^{\infty}\frac{1}{k2^n+1}.}



Λέξεις Κλειδιά:
Άβαταρ μέλους
Tolaso J Kos
Δημοσιεύσεις: 5267
Εγγραφή: Κυρ Αύγ 05, 2012 10:09 pm
Τοποθεσία: Λάρισα, Βαρκελώνη
Επικοινωνία:

Re: Putnam 2016/B6

#2

Μη αναγνωσμένη δημοσίευση από Tolaso J Kos » Τετ Ιαν 04, 2017 8:59 pm

Γεια σου Δημήτρη,

επειδή η διπλή σειρά συγκλίνει απόλυτα μπορούμε να εναλλάξουμε τη σειρά άθροισης. Τότε:
\displaystyle{\begin{aligned}  \sum_{k=1}^{\infty} \frac{(-1)^{k-1}}{k} \sum_{n=0}^{\infty} \frac{1}{k2^n+1} &= \sum_{k=1}^{\infty} \frac{(-1)^{k-1}}{k} \sum_{n=0}^{\infty} \int_{0}^{1} x^{k 2^n } \, {\rm d}x\\    
 &= \sum_{n=0}^{\infty} \int_{0}^{1}\sum_{k=1}^{\infty} \frac{(-1)^{k-1}}{k} x^{2^n k} \, {\rm d}x\\     
&= \sum_{n=0}^{\infty} \int_{0}^{1} \log \left ( 1+x^{2^n} \right ) \, {\rm d}x\\    
&= \int_{0}^{1}\sum_{n=0}^{\infty} \log \left ( 1+ x^{2^n} \right ) \, {\rm d}x  \\     
&= \int_{0}^{1} \log \prod_{n=0}^{\infty} \left ( 1+x^{2^n} \right ) \, {\rm d}x \\    
 &= - \int_{0}^{1} \log  \left ( 1-x \right ) \, {\rm d}x \\    
&=1  \end{aligned}} διότι
\displaystyle{\prod_{n=0}^{\infty} \left ( 1+ x^{2^n} \right ) = \left ( 1+x \right )\left ( 1+x^2 \right )\left ( 1+x^4 \right )\cdots = 1+x + x^2 + x^3 + \mathcal{O}\left ( x^4 \right ) = \sum_{n=0}^{\infty} x^n = \frac{1}{1-x}} Η παραπάνω τεχνική είναι πολύ γνωστή και υπάρχει σε διάφορα papers.


Η φαντασία είναι σημαντικότερη από τη γνώση !
\displaystyle{{\color{blue}\mathbf{Life=\int_{birth}^{death}\frac{happiness}{time}\Delta time} }}
nickolas tsik
Δημοσιεύσεις: 17
Εγγραφή: Σάβ Απρ 27, 2024 10:03 pm
Τοποθεσία: ΗΡΑΚΛΕΙΟ ΚΡΗΤΗΣ

Re: Putnam 2016/B6

#3

Μη αναγνωσμένη δημοσίευση από nickolas tsik » Πέμ Μάιος 02, 2024 1:49 pm

Καλησπέρα αν και παλιό το θρέντ και καταλαβαίνω τον τρόπο που προτείνει ο κ Tolsaso μπορείτε να μου πείτε αν το χάνω καπου στο παρακάτω επειδή βρίσκω το ίδιο;

Εστω Α==\sum_{k=1}^{\infty}\frac{(-1)^{k-1}}{k}\sum_{n=0}^{\infty}\frac{1}{k2^n+1}=\sum_{k=1}^{\infty}\frac{(-1)^{k-1}}{k}(\frac{1}{k+1}+\sum_{n=1}^{\infty}\frac{1}{k2^n+1})= \sum_{k=1}^{\infty}\frac{(-1)^{k-1}}{k(k+1)}+\sum_{k=1}^{\infty}\frac{(-1)^{k-1}}{k}\sum_{n=1}^{\infty}   
\frac{1}{k2^n+1}== = A+A_1    (1)


A_1=\sum_{k=1}^{\infty}\frac{(-1)^{k-1}}{k(k+1)}=\sum_{k=1}^{\infty}(-1)^{k-1}(\frac{1}{k}-\frac{1}{k+1})
Χωρίς να χαλάσουμε την γενικότητα έχουμε: =\lim_{z\rightarrow \infty}(\sum_{k=1}^{z}(-1)^{k-1}(\frac{1}{k}-\frac{1}{k+1})=\lim_{z\rightarrow \infty}\sum_{k=1}^{z}(\frac{(-1)^{k-1})}{k})-\sum_{k=1}^{z}\frac{(-1)^{k-1}}{k+1}= \lim_{z\rightarrow \infty}1+\sum_{k=2}^{z}\frac{(-1)^{k-1}}{k}-\frac{(-1)^{z}}{z+1}-\sum_{k=1}^{z-1}(\frac{(-1)^{k-1}}{k+1})...=\lim_{z\rightarrow \infty}1+\sum_{k=2}^{z}\frac{(-1)^{k-1}}{k}-\frac{(-1)^z}{z+1}+\sum_{k=2}^{z}\frac{(-1)^{k-1}}{k}...= \lim_{z\rightarrow \infty}(1+2\sum_{k=2}^{z})\frac{(-1)^{k-1}}{k}-\frac{(-1)^z}{z+1}...=-1+2\sum_{k=1}^{\infty}\frac{(-1)^{k-1}}{k}
Επειτα βλέπουμε οτι =-1+(1-\frac{1}{2}+\frac{1}{3}-\frac{1}{4}+.....)=-1+2ln2=2ln2-1. A_2=\sum_{k=1}^{\infty}\frac{(-1)^{k-1}}{k}\sum_{n=1}^{\infty}\frac{1}{k2^n+1}=\sum_{k=1}^{\infty}\frac{(-1)^{k-1}}{k}\sum_{n=1}^{\infty}\frac{1}{k2^n(1+(1+\frac{1}{k2^n}))}...\Leftrightarrow A_2=\sum_{k=1}^{\infty}\frac{(-1)^{k-1}}{k}\sum_{n=1}^{\infty}\frac{1}{k2^n}\sum_{m=0}^{\infty}\frac{(-1)^m}{k^m2^{nm}}
Με λίγες πράξεις βρήκα οτι =\sum_{m=0}^{\infty}(-1)^m\sum_{n=1}^{\infty}\frac{1}{2^{nm+n}}\sum_{k=1}^{\infty}\frac{(-1)^{k-1}}{k*k^{m+1}}=...=\sum_{m=0}^{\infty}(-1)^m(\frac{1}{2^{m+1}-1})\sum_{k=1}^{\infty}\frac{(-1)^{k-1}}{k^{m+2}}= \sum_{m=0}^{\infty}(-1)^m\frac{1}{2^{m+1}-1}A_3
\therefore A_3=\sum_{k=1}^{\infty}\frac{(-1)^{k-1}}{k^{m+2}} =(\frac{1}{1^{m+2}}+\frac{1}{3^{m+2}}+...)-(\frac{1}{2^{m+2}}+\frac{1}{4^{m+2}}+...)=...=(1+\frac{1}{2^{m+1}})\sum_{k+1}^{\infty}\frac{1}{k^{m+2}} ,\therefore A_2=\sum_{m=0}^{\infty}(\frac{1}{2^{m+1}-1})(1-\frac{1}{2^{m+1}})(\sum_{k=1}^{\infty})\frac{1}{k^{m+2}}=\sum_{mn=0}^{\infty}(-1)^m(\frac{1}{2^{m+1}-1})(\frac{2^{m+1}-1}{2^{m+1}})\sum_{k=1}^{\infty}\frac{1}{k^{m+1}}=...=\sum_{k=1}^{\infty}\frac{1}{2k^2}\sum_{m=0}^{\infty}\frac{(-1)^m}{K^m*2^m}=...=2\sum_{k=1}^{\infty}\frac{1}{2k(2k+1)}=2\sum_{k=1}^{\infty}(\frac{1}{2k}-\frac{1}{2k+1})=2(\frac{1}{2}-\frac{1}{3}+\frac{1}{4}-\frac{1}{5}+...+)=-2(\frac{-1}{2}+\frac{1}{3}-\frac{1}{4}+\frac{1}{5}-...)=2-ln2
Αρα μπορούμε να εφαρμόσουμε τις τιμές Α_1,Α_2 στην (1)\Rightarrow A=2ln2-1+2-2ln2=1
Επομέμως η απάντηση ειναι 1


Απάντηση

Επιστροφή σε “Διαγωνισμοί για φοιτητές”

Μέλη σε σύνδεση

Μέλη σε αυτήν τη Δ. Συζήτηση: Δεν υπάρχουν εγγεγραμμένα μέλη και 1 επισκέπτης